LSAT and Law School Admissions Forum

Get expert LSAT preparation and law school admissions advice from PowerScore Test Preparation.

 15veries
  • Posts: 113
  • Joined: Sep 25, 2016
|
#30843
Hi

So A is correct because in the first sentence it talks about negative correlation between the value of entertainment and caliber of the reporting and in the second sentence it introduces a new element "greatness" an connect it to an element in the first sentence, specifically "entertaining" (is the entertainment value=entertaining by the way?) And it concludes the critics are mistaken thinking both elements have positive correlation. This suggests the journalist treats the greatness and caliber are the same thing thus in this question the assumption is between the 2 premises....is this correct?
So I was not sure because I was not sure "value" is or is not a new element...so I was not sure how to eliminate C.
Is C wrong because it does not talk about "caliber"?
But then I thought caliber and greatness are the same thing actually...they mean how they superior in some way right?
Could anyone give explanations about A and C?

Thanks
 Adam Tyson
PowerScore Staff
  • PowerScore Staff
  • Posts: 5153
  • Joined: Apr 14, 2011
|
#31350
Your analysis of the stimulus and answer A are good (and yes, entertaining means it has entertainment value).

The problem with C is that it is too vague and does not connect clearly to the caliber of the reporting. The link between "great journalist" and "high caliber reporting" is what's missing, and because A makes that link it justifies the conclusion that the critics are mistaken. Answer C fails to make that link for us, instead linking to the more nebulous "valuable in some respect" claim. Maybe it's valuable because it makes us happy, or because it gives us a sense of security or comfort?

Don't make the leap that high caliber reporting is the same thing as great journalism, because we can't know that those two are identical in meaning. That's the missing link we need to provide, so don't just assume it. Justify the Conclusion answers can often be ones that make what seem like obvious connections, but without making the connection explicit the argument still lacks something. Don't shy away from what seem like obvious answers, so long as they close a logical gap for you.

Good work here! Keep it up!
 Margo
  • Posts: 18
  • Joined: Jun 25, 2018
|
#46922
Hello,

I had trouble with this question.

I know this is a justify question because of the question stem "if which one of the following is assumed."

I chose D because I thought that if entertainment and news were mutually exclusive categories, there would be no way for a reputable journalist delivering news to also be entertaining. Because as "entertainment value" in news increases, it is still news, otherwise the author's conclusion would not make sense.

I have trouble wrapping my head around A because I thought that it is not necessary that the news reporting of the greatest journalists have been of the highEST caliber. All that really matters is that the journalism is not at a LOWER level of caliber than news reporting in the past (the news reporting with less entertainment value). So, the journalism with more entertainment value could be of an EQUAL level of caliber of reporting, or slightly more, and the argument would still hold up ok.

Am I doing this wrong because I'm approaching it more like an assumption question rather than a justify question, and all I need is some evidence to strengthen the argument, which A does? I still have trouble ruling out D.

Thanks in advance :)
 Adam Tyson
PowerScore Staff
  • PowerScore Staff
  • Posts: 5153
  • Joined: Apr 14, 2011
|
#46990
This final analysis of yours is half right. half wrong, Margo:
Am I doing this wrong because I'm approaching it more like an assumption question rather than a justify question, and all I need is some evidence to strengthen the argument, which A does
It does look like you are treating this as an Assumption question, rather than a Justify the Conclusion question, including that you appear to be using a form of the Assumption Negation Technique on answer D. However, we need to do much more here than just find "some evidence to strengthen the argument", as that describes a Strengthen question.

The key to this being a Justify question is not the part of the stem that says "if which one of the following is assumed", because that is a part of lots of question stems, including weaken, strengthen, and even resolve the paradox. Rather, the key language in the stem is "The journalist’s conclusion is properly drawn", which means the conclusion absolutely must be true, or will be completely proven by, the correct answer.

If the greatest journalists do the highest caliber reporting, and if they are also entertaining, then that proves, with no room for doubt, that the folks who think quality goes down as entertainment goes up are wrong.

Answer D allows for the possibility of some news being entertaining, but doesn't prove that the folks who say there is an inverse correlation are wrong. Those folks would be fine with something being moderately newsworthy and also moderately entertaining. They would just say that as you increase the entertainment value, the news value will drop, and vice versa.

When asked to Justify a conclusion, find the answer that proves it, rather than just helping it! Keep at it, Margo!
 Margo
  • Posts: 18
  • Joined: Jun 25, 2018
|
#47007
Thank you! This explanation is so helpful! :-D

Get the most out of your LSAT Prep Plus subscription.

Analyze and track your performance with our Testing and Analytics Package.